Prove the alternating sum of a decreasing sequence converging to 0 is Cauchy. Unicorn Meta Zoo #1: Why another podcast? Announcing the arrival of Valued Associate #679: Cesar ManaraSuppose for all positive integers $n$, $|x_n-y_n|< frac1n$ Prove that $(x_n)$ is also Cauchy.Proof check for completenessProve that $d_n$ is a Cauchy sequence in $mathbbR$Prove $aX_n +bY_n$ is a Cauchy Sequence.Prove a sequence is a Cauchy and thus convergentIf $(x_n)$ and $(y_n)$ are Cauchy sequences, then give a direct argument that $ (x_n + y_n)$ is a Cauchy sequenceIf $x_n$ and $y_n$ are Cauchy then $leftfrac2x_ny_nright$ is CauchyLet $x_n$ be a Cauchy sequence of rational numbers. Define a new sequence $y_n$ by $y_n = (x_n)(x_n+1)$. Show that $y_n$ is a CS.Let $x_n$ be a Cauchy sequence of real numbers, prove that a new sequence $y_n$, with $y_n$=$x_n^frac13$, is also a Cauchy sequence.$x_n rightarrow x$ iff the modified sequence is Cauchy

"Whatever a Russian does, they end up making the Kalashnikov gun"? Are there any similar proverbs in English?

How can I wire a 9-position switch so that each position turns on one more LED than the one before?

Is Diceware more secure than a long passphrase?

How would this chord from "Rocket Man" be analyzed?

Trumpet valves, lengths, and pitch

All ASCII characters with a given bit count

What's the difference between using dependency injection with a container and using a service locator?

What is the term for a person whose job is to place products on shelves in stores?

Did the Roman Empire have penal colonies?

"My boss was furious with me and I have been fired" vs. "My boss was furious with me and I was fired"

My admission is revoked after accepting the admission offer

Does Feeblemind produce an ongoing magical effect that can be dispelled?

Mistake in years of experience in resume?

What is the best way to deal with NPC-NPC combat?

Justification for leaving new position after a short time

I preordered a game on my Xbox while on the home screen of my friend's account. Which of us owns the game?

Putting Ant-Man on house arrest

How to keep bees out of canned beverages?

Is accepting an invalid credit card number a security issue?

c++ diamond problem - How to call base method only once

Reattaching fallen shelf to wall?

Why didn't the Space Shuttle bounce back into space as many times as possible so as to lose a lot of kinetic energy up there?

What ability score does a Hexblade's Pact Weapon use for attack and damage when wielded by another character?

Co-worker works way more than he should



Prove the alternating sum of a decreasing sequence converging to 0 is Cauchy.



Unicorn Meta Zoo #1: Why another podcast?
Announcing the arrival of Valued Associate #679: Cesar ManaraSuppose for all positive integers $n$, $|x_n-y_n|< frac1n$ Prove that $(x_n)$ is also Cauchy.Proof check for completenessProve that $d_n$ is a Cauchy sequence in $mathbbR$Prove $aX_n +bY_n$ is a Cauchy Sequence.Prove a sequence is a Cauchy and thus convergentIf $(x_n)$ and $(y_n)$ are Cauchy sequences, then give a direct argument that $ (x_n + y_n)$ is a Cauchy sequenceIf $x_n$ and $y_n$ are Cauchy then $leftfrac2x_ny_nright$ is CauchyLet $x_n$ be a Cauchy sequence of rational numbers. Define a new sequence $y_n$ by $y_n = (x_n)(x_n+1)$. Show that $y_n$ is a CS.Let $x_n$ be a Cauchy sequence of real numbers, prove that a new sequence $y_n$, with $y_n$=$x_n^frac13$, is also a Cauchy sequence.$x_n rightarrow x$ iff the modified sequence is Cauchy










2












$begingroup$


Let $(x_n)$ be a decreasing sequence with $x_n > 0$ for all $n in mathbbN$, and $(x_n) to 0$. Let $(y_n)$ be defined for all $n in mathbbN$ by
$$y_n = x_0 - x_1 + x_2 - cdots + (-1)^n x_n .$$



I want to show, using the $varepsilon$ definition, that $(y_n)$ is Cauchy.



I am trying to find, given $varepsilon > 0$, a real number $N$ such that for all $m$ and $n$ with $m > n > N$, $|y_m - y_n| < varepsilon$.



I have been going backwards to try and find $N$, and have
beginalign*
|y_m - y_n| & = left| (x_0 - x_1 + cdots pm x_m) - (x_0 - x_1 + cdots pm x_n) right| \
|y_m - y_n| & = left| x_n + 1 - x_n + 2 + cdots pm x_m right| \
|y_m - y_n| & leq | x_n + 1 | + | x_n + 2 | + cdots + | x_m | \
|y_m - y_n| & leq ?
endalign*



I do not know how to get a solution from there, and am not sure about the process, particurlary the last step since I feel getting rid of the minuses might prevent me from finding a solution.










share|cite|improve this question









$endgroup$







  • 1




    $begingroup$
    Because the series is alternating and decreasing, I think you can prove by induction on $m$ that $|y_m-y_n| leq |y_n|$.
    $endgroup$
    – Robert Shore
    3 hours ago










  • $begingroup$
    @RobertShore is my answer okay?
    $endgroup$
    – Subhasis Biswas
    3 hours ago










  • $begingroup$
    @RobertShore yes I can definitely show that, but it brings me to the same issue with $|y_m| leq |x_0 - x_1 + cdots pm x_m|$, and I am unsure how to proceed from there.
    $endgroup$
    – oranji
    1 hour ago
















2












$begingroup$


Let $(x_n)$ be a decreasing sequence with $x_n > 0$ for all $n in mathbbN$, and $(x_n) to 0$. Let $(y_n)$ be defined for all $n in mathbbN$ by
$$y_n = x_0 - x_1 + x_2 - cdots + (-1)^n x_n .$$



I want to show, using the $varepsilon$ definition, that $(y_n)$ is Cauchy.



I am trying to find, given $varepsilon > 0$, a real number $N$ such that for all $m$ and $n$ with $m > n > N$, $|y_m - y_n| < varepsilon$.



I have been going backwards to try and find $N$, and have
beginalign*
|y_m - y_n| & = left| (x_0 - x_1 + cdots pm x_m) - (x_0 - x_1 + cdots pm x_n) right| \
|y_m - y_n| & = left| x_n + 1 - x_n + 2 + cdots pm x_m right| \
|y_m - y_n| & leq | x_n + 1 | + | x_n + 2 | + cdots + | x_m | \
|y_m - y_n| & leq ?
endalign*



I do not know how to get a solution from there, and am not sure about the process, particurlary the last step since I feel getting rid of the minuses might prevent me from finding a solution.










share|cite|improve this question









$endgroup$







  • 1




    $begingroup$
    Because the series is alternating and decreasing, I think you can prove by induction on $m$ that $|y_m-y_n| leq |y_n|$.
    $endgroup$
    – Robert Shore
    3 hours ago










  • $begingroup$
    @RobertShore is my answer okay?
    $endgroup$
    – Subhasis Biswas
    3 hours ago










  • $begingroup$
    @RobertShore yes I can definitely show that, but it brings me to the same issue with $|y_m| leq |x_0 - x_1 + cdots pm x_m|$, and I am unsure how to proceed from there.
    $endgroup$
    – oranji
    1 hour ago














2












2








2


1



$begingroup$


Let $(x_n)$ be a decreasing sequence with $x_n > 0$ for all $n in mathbbN$, and $(x_n) to 0$. Let $(y_n)$ be defined for all $n in mathbbN$ by
$$y_n = x_0 - x_1 + x_2 - cdots + (-1)^n x_n .$$



I want to show, using the $varepsilon$ definition, that $(y_n)$ is Cauchy.



I am trying to find, given $varepsilon > 0$, a real number $N$ such that for all $m$ and $n$ with $m > n > N$, $|y_m - y_n| < varepsilon$.



I have been going backwards to try and find $N$, and have
beginalign*
|y_m - y_n| & = left| (x_0 - x_1 + cdots pm x_m) - (x_0 - x_1 + cdots pm x_n) right| \
|y_m - y_n| & = left| x_n + 1 - x_n + 2 + cdots pm x_m right| \
|y_m - y_n| & leq | x_n + 1 | + | x_n + 2 | + cdots + | x_m | \
|y_m - y_n| & leq ?
endalign*



I do not know how to get a solution from there, and am not sure about the process, particurlary the last step since I feel getting rid of the minuses might prevent me from finding a solution.










share|cite|improve this question









$endgroup$




Let $(x_n)$ be a decreasing sequence with $x_n > 0$ for all $n in mathbbN$, and $(x_n) to 0$. Let $(y_n)$ be defined for all $n in mathbbN$ by
$$y_n = x_0 - x_1 + x_2 - cdots + (-1)^n x_n .$$



I want to show, using the $varepsilon$ definition, that $(y_n)$ is Cauchy.



I am trying to find, given $varepsilon > 0$, a real number $N$ such that for all $m$ and $n$ with $m > n > N$, $|y_m - y_n| < varepsilon$.



I have been going backwards to try and find $N$, and have
beginalign*
|y_m - y_n| & = left| (x_0 - x_1 + cdots pm x_m) - (x_0 - x_1 + cdots pm x_n) right| \
|y_m - y_n| & = left| x_n + 1 - x_n + 2 + cdots pm x_m right| \
|y_m - y_n| & leq | x_n + 1 | + | x_n + 2 | + cdots + | x_m | \
|y_m - y_n| & leq ?
endalign*



I do not know how to get a solution from there, and am not sure about the process, particurlary the last step since I feel getting rid of the minuses might prevent me from finding a solution.







real-analysis cauchy-sequences






share|cite|improve this question













share|cite|improve this question











share|cite|improve this question




share|cite|improve this question










asked 4 hours ago









oranjioranji

616




616







  • 1




    $begingroup$
    Because the series is alternating and decreasing, I think you can prove by induction on $m$ that $|y_m-y_n| leq |y_n|$.
    $endgroup$
    – Robert Shore
    3 hours ago










  • $begingroup$
    @RobertShore is my answer okay?
    $endgroup$
    – Subhasis Biswas
    3 hours ago










  • $begingroup$
    @RobertShore yes I can definitely show that, but it brings me to the same issue with $|y_m| leq |x_0 - x_1 + cdots pm x_m|$, and I am unsure how to proceed from there.
    $endgroup$
    – oranji
    1 hour ago













  • 1




    $begingroup$
    Because the series is alternating and decreasing, I think you can prove by induction on $m$ that $|y_m-y_n| leq |y_n|$.
    $endgroup$
    – Robert Shore
    3 hours ago










  • $begingroup$
    @RobertShore is my answer okay?
    $endgroup$
    – Subhasis Biswas
    3 hours ago










  • $begingroup$
    @RobertShore yes I can definitely show that, but it brings me to the same issue with $|y_m| leq |x_0 - x_1 + cdots pm x_m|$, and I am unsure how to proceed from there.
    $endgroup$
    – oranji
    1 hour ago








1




1




$begingroup$
Because the series is alternating and decreasing, I think you can prove by induction on $m$ that $|y_m-y_n| leq |y_n|$.
$endgroup$
– Robert Shore
3 hours ago




$begingroup$
Because the series is alternating and decreasing, I think you can prove by induction on $m$ that $|y_m-y_n| leq |y_n|$.
$endgroup$
– Robert Shore
3 hours ago












$begingroup$
@RobertShore is my answer okay?
$endgroup$
– Subhasis Biswas
3 hours ago




$begingroup$
@RobertShore is my answer okay?
$endgroup$
– Subhasis Biswas
3 hours ago












$begingroup$
@RobertShore yes I can definitely show that, but it brings me to the same issue with $|y_m| leq |x_0 - x_1 + cdots pm x_m|$, and I am unsure how to proceed from there.
$endgroup$
– oranji
1 hour ago





$begingroup$
@RobertShore yes I can definitely show that, but it brings me to the same issue with $|y_m| leq |x_0 - x_1 + cdots pm x_m|$, and I am unsure how to proceed from there.
$endgroup$
– oranji
1 hour ago











2 Answers
2






active

oldest

votes


















2












$begingroup$

To see that the sequence of partial sums is Cauchy, you cannot use the triangle inequality directly as you did. A famous counter example here is $sum_k=1^inftyfrac(-1)^kk$.



What you can do is grouping the terms of the partial sums $s_n= sum_j=1^n(-1)^jx_j$ as follows:



  • Let $m = n+k, k,n in mathbbN$

Now, you can write $|s_m - s_n|$ in two different ways:



$$|s_n+k - s_n| = begincases
|x_n+1 - (x_n+2-x_n+3) - cdots - (x_n+2i-x_n+2i+1)| & k = 2i+1 \
|x_n+1 - (x_n+2-x_n+3) - cdots - (x_n+2i-2-x_n+2i-1) - x_2i| & k = 2i \
endcases
$$



$$|s_n+k - s_n| = begincases
|(x_n+1 - x_n+2) + cdots + (x_n+2i-1-x_n+2i) + x_n+2i+1| & k = 2i+1 \
|(x_n+1 - x_n+2) + cdots + (x_n+2i-1-x_n+2i) | & k = 2i \
endcases
$$



Using the fact that $x_n searrow 0$, it follows immediately that for all $k in mathbbN$ holds
$$|s_n+k - s_n| leq x_n+1$$



Hence, for $epsilon > 0$ choose $N_epsilon$ such that $x_N_epsilon < epsilon$. Then, for all $m> n > N_epsilon$ you have
$$|s_m - s_n| leq x_n+1 leq x_N_epsilon < epsilon$$






share|cite|improve this answer











$endgroup$












  • $begingroup$
    This is exactly what I was about to do.
    $endgroup$
    – Subhasis Biswas
    1 hour ago






  • 1




    $begingroup$
    @SubhasisBiswas So, I did it for you :-D
    $endgroup$
    – trancelocation
    1 hour ago


















2












$begingroup$

This is also known as the "Leibnitz's Test".



We write $s_n = x_1-x_2+x_3-...+(-1)^n+1x_n$



$s_2n+2-s_2n=u_2n+1-u_2n+2 geq0$ for all $n$.



$s_2n+1-s_2n-1=-u_2n+u_2n+1 leq 0$



$s_2n =u_1 -(u_2-u_3)-(u_4-u_5)...-u_2n leq u_1$, i.e. a monotone increasing sequence bounded above.



$s_2n+1 =(u_1 -u_2)+(u_3-u_4)+...+u_2n+1 geq u_1-u_2$, i.e. a monotone decreasing sequence bounded below.



Hence, both are convergent subsequences of $(s_n)$. But, we have $lim (s_2n+1-s_2n)=u_2n+1=0$, therefore, they converge to the same limit.



Hence, $(s_n)$ converges, i.e. it is Cauchy.



Note: We conclude that $(s_n)$ converges because the indices of the two subsequences $(s_2n)$ and $(s_2n+1)$ i.e. $U = 2n+1 : n in mathbbN$ and $V = 2n : n in mathbbN$ form a partition of $mathbbN$ and they both converge to the same limit.






share|cite|improve this answer









$endgroup$












  • $begingroup$
    I want to use the $varepsilon$ definition of a Cauchy sequence, and not the fact that all convergent sequences are Cauchy, which is why I cannot use this solution.
    $endgroup$
    – oranji
    1 hour ago










  • $begingroup$
    I'll edit this answer.
    $endgroup$
    – Subhasis Biswas
    1 hour ago











Your Answer








StackExchange.ready(function()
var channelOptions =
tags: "".split(" "),
id: "69"
;
initTagRenderer("".split(" "), "".split(" "), channelOptions);

StackExchange.using("externalEditor", function()
// Have to fire editor after snippets, if snippets enabled
if (StackExchange.settings.snippets.snippetsEnabled)
StackExchange.using("snippets", function()
createEditor();
);

else
createEditor();

);

function createEditor()
StackExchange.prepareEditor(
heartbeatType: 'answer',
autoActivateHeartbeat: false,
convertImagesToLinks: true,
noModals: true,
showLowRepImageUploadWarning: true,
reputationToPostImages: 10,
bindNavPrevention: true,
postfix: "",
imageUploader:
brandingHtml: "Powered by u003ca class="icon-imgur-white" href="https://imgur.com/"u003eu003c/au003e",
contentPolicyHtml: "User contributions licensed under u003ca href="https://creativecommons.org/licenses/by-sa/3.0/"u003ecc by-sa 3.0 with attribution requiredu003c/au003e u003ca href="https://stackoverflow.com/legal/content-policy"u003e(content policy)u003c/au003e",
allowUrls: true
,
noCode: true, onDemand: true,
discardSelector: ".discard-answer"
,immediatelyShowMarkdownHelp:true
);



);













draft saved

draft discarded


















StackExchange.ready(
function ()
StackExchange.openid.initPostLogin('.new-post-login', 'https%3a%2f%2fmath.stackexchange.com%2fquestions%2f3201256%2fprove-the-alternating-sum-of-a-decreasing-sequence-converging-to-0-is-cauchy%23new-answer', 'question_page');

);

Post as a guest















Required, but never shown

























2 Answers
2






active

oldest

votes








2 Answers
2






active

oldest

votes









active

oldest

votes






active

oldest

votes









2












$begingroup$

To see that the sequence of partial sums is Cauchy, you cannot use the triangle inequality directly as you did. A famous counter example here is $sum_k=1^inftyfrac(-1)^kk$.



What you can do is grouping the terms of the partial sums $s_n= sum_j=1^n(-1)^jx_j$ as follows:



  • Let $m = n+k, k,n in mathbbN$

Now, you can write $|s_m - s_n|$ in two different ways:



$$|s_n+k - s_n| = begincases
|x_n+1 - (x_n+2-x_n+3) - cdots - (x_n+2i-x_n+2i+1)| & k = 2i+1 \
|x_n+1 - (x_n+2-x_n+3) - cdots - (x_n+2i-2-x_n+2i-1) - x_2i| & k = 2i \
endcases
$$



$$|s_n+k - s_n| = begincases
|(x_n+1 - x_n+2) + cdots + (x_n+2i-1-x_n+2i) + x_n+2i+1| & k = 2i+1 \
|(x_n+1 - x_n+2) + cdots + (x_n+2i-1-x_n+2i) | & k = 2i \
endcases
$$



Using the fact that $x_n searrow 0$, it follows immediately that for all $k in mathbbN$ holds
$$|s_n+k - s_n| leq x_n+1$$



Hence, for $epsilon > 0$ choose $N_epsilon$ such that $x_N_epsilon < epsilon$. Then, for all $m> n > N_epsilon$ you have
$$|s_m - s_n| leq x_n+1 leq x_N_epsilon < epsilon$$






share|cite|improve this answer











$endgroup$












  • $begingroup$
    This is exactly what I was about to do.
    $endgroup$
    – Subhasis Biswas
    1 hour ago






  • 1




    $begingroup$
    @SubhasisBiswas So, I did it for you :-D
    $endgroup$
    – trancelocation
    1 hour ago















2












$begingroup$

To see that the sequence of partial sums is Cauchy, you cannot use the triangle inequality directly as you did. A famous counter example here is $sum_k=1^inftyfrac(-1)^kk$.



What you can do is grouping the terms of the partial sums $s_n= sum_j=1^n(-1)^jx_j$ as follows:



  • Let $m = n+k, k,n in mathbbN$

Now, you can write $|s_m - s_n|$ in two different ways:



$$|s_n+k - s_n| = begincases
|x_n+1 - (x_n+2-x_n+3) - cdots - (x_n+2i-x_n+2i+1)| & k = 2i+1 \
|x_n+1 - (x_n+2-x_n+3) - cdots - (x_n+2i-2-x_n+2i-1) - x_2i| & k = 2i \
endcases
$$



$$|s_n+k - s_n| = begincases
|(x_n+1 - x_n+2) + cdots + (x_n+2i-1-x_n+2i) + x_n+2i+1| & k = 2i+1 \
|(x_n+1 - x_n+2) + cdots + (x_n+2i-1-x_n+2i) | & k = 2i \
endcases
$$



Using the fact that $x_n searrow 0$, it follows immediately that for all $k in mathbbN$ holds
$$|s_n+k - s_n| leq x_n+1$$



Hence, for $epsilon > 0$ choose $N_epsilon$ such that $x_N_epsilon < epsilon$. Then, for all $m> n > N_epsilon$ you have
$$|s_m - s_n| leq x_n+1 leq x_N_epsilon < epsilon$$






share|cite|improve this answer











$endgroup$












  • $begingroup$
    This is exactly what I was about to do.
    $endgroup$
    – Subhasis Biswas
    1 hour ago






  • 1




    $begingroup$
    @SubhasisBiswas So, I did it for you :-D
    $endgroup$
    – trancelocation
    1 hour ago













2












2








2





$begingroup$

To see that the sequence of partial sums is Cauchy, you cannot use the triangle inequality directly as you did. A famous counter example here is $sum_k=1^inftyfrac(-1)^kk$.



What you can do is grouping the terms of the partial sums $s_n= sum_j=1^n(-1)^jx_j$ as follows:



  • Let $m = n+k, k,n in mathbbN$

Now, you can write $|s_m - s_n|$ in two different ways:



$$|s_n+k - s_n| = begincases
|x_n+1 - (x_n+2-x_n+3) - cdots - (x_n+2i-x_n+2i+1)| & k = 2i+1 \
|x_n+1 - (x_n+2-x_n+3) - cdots - (x_n+2i-2-x_n+2i-1) - x_2i| & k = 2i \
endcases
$$



$$|s_n+k - s_n| = begincases
|(x_n+1 - x_n+2) + cdots + (x_n+2i-1-x_n+2i) + x_n+2i+1| & k = 2i+1 \
|(x_n+1 - x_n+2) + cdots + (x_n+2i-1-x_n+2i) | & k = 2i \
endcases
$$



Using the fact that $x_n searrow 0$, it follows immediately that for all $k in mathbbN$ holds
$$|s_n+k - s_n| leq x_n+1$$



Hence, for $epsilon > 0$ choose $N_epsilon$ such that $x_N_epsilon < epsilon$. Then, for all $m> n > N_epsilon$ you have
$$|s_m - s_n| leq x_n+1 leq x_N_epsilon < epsilon$$






share|cite|improve this answer











$endgroup$



To see that the sequence of partial sums is Cauchy, you cannot use the triangle inequality directly as you did. A famous counter example here is $sum_k=1^inftyfrac(-1)^kk$.



What you can do is grouping the terms of the partial sums $s_n= sum_j=1^n(-1)^jx_j$ as follows:



  • Let $m = n+k, k,n in mathbbN$

Now, you can write $|s_m - s_n|$ in two different ways:



$$|s_n+k - s_n| = begincases
|x_n+1 - (x_n+2-x_n+3) - cdots - (x_n+2i-x_n+2i+1)| & k = 2i+1 \
|x_n+1 - (x_n+2-x_n+3) - cdots - (x_n+2i-2-x_n+2i-1) - x_2i| & k = 2i \
endcases
$$



$$|s_n+k - s_n| = begincases
|(x_n+1 - x_n+2) + cdots + (x_n+2i-1-x_n+2i) + x_n+2i+1| & k = 2i+1 \
|(x_n+1 - x_n+2) + cdots + (x_n+2i-1-x_n+2i) | & k = 2i \
endcases
$$



Using the fact that $x_n searrow 0$, it follows immediately that for all $k in mathbbN$ holds
$$|s_n+k - s_n| leq x_n+1$$



Hence, for $epsilon > 0$ choose $N_epsilon$ such that $x_N_epsilon < epsilon$. Then, for all $m> n > N_epsilon$ you have
$$|s_m - s_n| leq x_n+1 leq x_N_epsilon < epsilon$$







share|cite|improve this answer














share|cite|improve this answer



share|cite|improve this answer








edited 46 mins ago

























answered 1 hour ago









trancelocationtrancelocation

14.6k1929




14.6k1929











  • $begingroup$
    This is exactly what I was about to do.
    $endgroup$
    – Subhasis Biswas
    1 hour ago






  • 1




    $begingroup$
    @SubhasisBiswas So, I did it for you :-D
    $endgroup$
    – trancelocation
    1 hour ago
















  • $begingroup$
    This is exactly what I was about to do.
    $endgroup$
    – Subhasis Biswas
    1 hour ago






  • 1




    $begingroup$
    @SubhasisBiswas So, I did it for you :-D
    $endgroup$
    – trancelocation
    1 hour ago















$begingroup$
This is exactly what I was about to do.
$endgroup$
– Subhasis Biswas
1 hour ago




$begingroup$
This is exactly what I was about to do.
$endgroup$
– Subhasis Biswas
1 hour ago




1




1




$begingroup$
@SubhasisBiswas So, I did it for you :-D
$endgroup$
– trancelocation
1 hour ago




$begingroup$
@SubhasisBiswas So, I did it for you :-D
$endgroup$
– trancelocation
1 hour ago











2












$begingroup$

This is also known as the "Leibnitz's Test".



We write $s_n = x_1-x_2+x_3-...+(-1)^n+1x_n$



$s_2n+2-s_2n=u_2n+1-u_2n+2 geq0$ for all $n$.



$s_2n+1-s_2n-1=-u_2n+u_2n+1 leq 0$



$s_2n =u_1 -(u_2-u_3)-(u_4-u_5)...-u_2n leq u_1$, i.e. a monotone increasing sequence bounded above.



$s_2n+1 =(u_1 -u_2)+(u_3-u_4)+...+u_2n+1 geq u_1-u_2$, i.e. a monotone decreasing sequence bounded below.



Hence, both are convergent subsequences of $(s_n)$. But, we have $lim (s_2n+1-s_2n)=u_2n+1=0$, therefore, they converge to the same limit.



Hence, $(s_n)$ converges, i.e. it is Cauchy.



Note: We conclude that $(s_n)$ converges because the indices of the two subsequences $(s_2n)$ and $(s_2n+1)$ i.e. $U = 2n+1 : n in mathbbN$ and $V = 2n : n in mathbbN$ form a partition of $mathbbN$ and they both converge to the same limit.






share|cite|improve this answer









$endgroup$












  • $begingroup$
    I want to use the $varepsilon$ definition of a Cauchy sequence, and not the fact that all convergent sequences are Cauchy, which is why I cannot use this solution.
    $endgroup$
    – oranji
    1 hour ago










  • $begingroup$
    I'll edit this answer.
    $endgroup$
    – Subhasis Biswas
    1 hour ago















2












$begingroup$

This is also known as the "Leibnitz's Test".



We write $s_n = x_1-x_2+x_3-...+(-1)^n+1x_n$



$s_2n+2-s_2n=u_2n+1-u_2n+2 geq0$ for all $n$.



$s_2n+1-s_2n-1=-u_2n+u_2n+1 leq 0$



$s_2n =u_1 -(u_2-u_3)-(u_4-u_5)...-u_2n leq u_1$, i.e. a monotone increasing sequence bounded above.



$s_2n+1 =(u_1 -u_2)+(u_3-u_4)+...+u_2n+1 geq u_1-u_2$, i.e. a monotone decreasing sequence bounded below.



Hence, both are convergent subsequences of $(s_n)$. But, we have $lim (s_2n+1-s_2n)=u_2n+1=0$, therefore, they converge to the same limit.



Hence, $(s_n)$ converges, i.e. it is Cauchy.



Note: We conclude that $(s_n)$ converges because the indices of the two subsequences $(s_2n)$ and $(s_2n+1)$ i.e. $U = 2n+1 : n in mathbbN$ and $V = 2n : n in mathbbN$ form a partition of $mathbbN$ and they both converge to the same limit.






share|cite|improve this answer









$endgroup$












  • $begingroup$
    I want to use the $varepsilon$ definition of a Cauchy sequence, and not the fact that all convergent sequences are Cauchy, which is why I cannot use this solution.
    $endgroup$
    – oranji
    1 hour ago










  • $begingroup$
    I'll edit this answer.
    $endgroup$
    – Subhasis Biswas
    1 hour ago













2












2








2





$begingroup$

This is also known as the "Leibnitz's Test".



We write $s_n = x_1-x_2+x_3-...+(-1)^n+1x_n$



$s_2n+2-s_2n=u_2n+1-u_2n+2 geq0$ for all $n$.



$s_2n+1-s_2n-1=-u_2n+u_2n+1 leq 0$



$s_2n =u_1 -(u_2-u_3)-(u_4-u_5)...-u_2n leq u_1$, i.e. a monotone increasing sequence bounded above.



$s_2n+1 =(u_1 -u_2)+(u_3-u_4)+...+u_2n+1 geq u_1-u_2$, i.e. a monotone decreasing sequence bounded below.



Hence, both are convergent subsequences of $(s_n)$. But, we have $lim (s_2n+1-s_2n)=u_2n+1=0$, therefore, they converge to the same limit.



Hence, $(s_n)$ converges, i.e. it is Cauchy.



Note: We conclude that $(s_n)$ converges because the indices of the two subsequences $(s_2n)$ and $(s_2n+1)$ i.e. $U = 2n+1 : n in mathbbN$ and $V = 2n : n in mathbbN$ form a partition of $mathbbN$ and they both converge to the same limit.






share|cite|improve this answer









$endgroup$



This is also known as the "Leibnitz's Test".



We write $s_n = x_1-x_2+x_3-...+(-1)^n+1x_n$



$s_2n+2-s_2n=u_2n+1-u_2n+2 geq0$ for all $n$.



$s_2n+1-s_2n-1=-u_2n+u_2n+1 leq 0$



$s_2n =u_1 -(u_2-u_3)-(u_4-u_5)...-u_2n leq u_1$, i.e. a monotone increasing sequence bounded above.



$s_2n+1 =(u_1 -u_2)+(u_3-u_4)+...+u_2n+1 geq u_1-u_2$, i.e. a monotone decreasing sequence bounded below.



Hence, both are convergent subsequences of $(s_n)$. But, we have $lim (s_2n+1-s_2n)=u_2n+1=0$, therefore, they converge to the same limit.



Hence, $(s_n)$ converges, i.e. it is Cauchy.



Note: We conclude that $(s_n)$ converges because the indices of the two subsequences $(s_2n)$ and $(s_2n+1)$ i.e. $U = 2n+1 : n in mathbbN$ and $V = 2n : n in mathbbN$ form a partition of $mathbbN$ and they both converge to the same limit.







share|cite|improve this answer












share|cite|improve this answer



share|cite|improve this answer










answered 3 hours ago









Subhasis BiswasSubhasis Biswas

608512




608512











  • $begingroup$
    I want to use the $varepsilon$ definition of a Cauchy sequence, and not the fact that all convergent sequences are Cauchy, which is why I cannot use this solution.
    $endgroup$
    – oranji
    1 hour ago










  • $begingroup$
    I'll edit this answer.
    $endgroup$
    – Subhasis Biswas
    1 hour ago
















  • $begingroup$
    I want to use the $varepsilon$ definition of a Cauchy sequence, and not the fact that all convergent sequences are Cauchy, which is why I cannot use this solution.
    $endgroup$
    – oranji
    1 hour ago










  • $begingroup$
    I'll edit this answer.
    $endgroup$
    – Subhasis Biswas
    1 hour ago















$begingroup$
I want to use the $varepsilon$ definition of a Cauchy sequence, and not the fact that all convergent sequences are Cauchy, which is why I cannot use this solution.
$endgroup$
– oranji
1 hour ago




$begingroup$
I want to use the $varepsilon$ definition of a Cauchy sequence, and not the fact that all convergent sequences are Cauchy, which is why I cannot use this solution.
$endgroup$
– oranji
1 hour ago












$begingroup$
I'll edit this answer.
$endgroup$
– Subhasis Biswas
1 hour ago




$begingroup$
I'll edit this answer.
$endgroup$
– Subhasis Biswas
1 hour ago

















draft saved

draft discarded
















































Thanks for contributing an answer to Mathematics Stack Exchange!


  • Please be sure to answer the question. Provide details and share your research!

But avoid


  • Asking for help, clarification, or responding to other answers.

  • Making statements based on opinion; back them up with references or personal experience.

Use MathJax to format equations. MathJax reference.


To learn more, see our tips on writing great answers.




draft saved


draft discarded














StackExchange.ready(
function ()
StackExchange.openid.initPostLogin('.new-post-login', 'https%3a%2f%2fmath.stackexchange.com%2fquestions%2f3201256%2fprove-the-alternating-sum-of-a-decreasing-sequence-converging-to-0-is-cauchy%23new-answer', 'question_page');

);

Post as a guest















Required, but never shown





















































Required, but never shown














Required, but never shown












Required, but never shown







Required, but never shown

































Required, but never shown














Required, but never shown












Required, but never shown







Required, but never shown







Popular posts from this blog

Dapidodigma demeter Subspecies | Notae | Tabula navigationisDapidodigmaAfrotropical Butterflies: Lycaenidae - Subtribe IolainaAmplifica

Constantinus Vanšenkin Nexus externi | Tabula navigationisБольшая российская энциклопедияAmplifica

Vas sanguineum Index Historia | Divisio | Constructio anatomica | Vasorum sanguineorum morbi (angiopathiae) | Notae | Nexus interniTabula navigationisAmplifica